|
|
Line 11: |
Line 11: |
| '''''Note''': A permutation of the set <math> \{ 1, 2, \ldots, n \} </math> is a one-to-one function of this set to itself.'' | | '''''Note''': A permutation of the set <math> \{ 1, 2, \ldots, n \} </math> is a one-to-one function of this set to itself.'' |
| | | |
− | == Solution 1 == | + | == Solution == |
| The only solutions are <math>n=1 </math> and <math>n=3 </math>. | | The only solutions are <math>n=1 </math> and <math>n=3 </math>. |
| | | |
Line 31: |
Line 31: |
| | | |
| We now have the cases <math>n=1,2,3,4 </math> to consider. The case <math>n=1 </math> is trivial. For <math>n=2 </math> it is easy to verify that neither <math> \sqrt{2 + \sqrt{1}} </math> nor <math> \sqrt{1 + \sqrt{2}} </math> is rational. For <math>n=3 </math>, we have the solution <math> \sqrt{ 2 + \sqrt{ 3 + \sqrt{1}}} = 2 </math>. We now consider. <math>n=4 </math>. First, suppose <math>4 \neq \sigma(4) </math>; say <math>\sigma(i) = 4 </math>. We have already shown that <math> x= \sqrt{ \sigma(i+1) + \sqrt{ \cdots + \sqrt{\sigma(4)}}} \le 2 < 5 </math>; this means <math>4 < \sigma(i)+x <9 </math>, a contradiction, since <math>\sigma(i) + x </math> must be a perfect square. Thus <math>\sigma(4) =4 </math>. But here we have either <math> \sqrt{1 + \sqrt{4}} </math> is irrational, <math> \sqrt{3 + \sqrt{4}} </math> is irrational, <math> \sqrt{1 + \sqrt{2 + \sqrt{4}}} </math> is irrational, or <math> \sqrt{3 + \sqrt{2 + \sqrt{4}}} </math> is irrational. Thus there is no solution for <math>n=4 </math> and the only solutions are <math>n=1, 3 </math>. | | We now have the cases <math>n=1,2,3,4 </math> to consider. The case <math>n=1 </math> is trivial. For <math>n=2 </math> it is easy to verify that neither <math> \sqrt{2 + \sqrt{1}} </math> nor <math> \sqrt{1 + \sqrt{2}} </math> is rational. For <math>n=3 </math>, we have the solution <math> \sqrt{ 2 + \sqrt{ 3 + \sqrt{1}}} = 2 </math>. We now consider. <math>n=4 </math>. First, suppose <math>4 \neq \sigma(4) </math>; say <math>\sigma(i) = 4 </math>. We have already shown that <math> x= \sqrt{ \sigma(i+1) + \sqrt{ \cdots + \sqrt{\sigma(4)}}} \le 2 < 5 </math>; this means <math>4 < \sigma(i)+x <9 </math>, a contradiction, since <math>\sigma(i) + x </math> must be a perfect square. Thus <math>\sigma(4) =4 </math>. But here we have either <math> \sqrt{1 + \sqrt{4}} </math> is irrational, <math> \sqrt{3 + \sqrt{4}} </math> is irrational, <math> \sqrt{1 + \sqrt{2 + \sqrt{4}}} </math> is irrational, or <math> \sqrt{3 + \sqrt{2 + \sqrt{4}}} </math> is irrational. Thus there is no solution for <math>n=4 </math> and the only solutions are <math>n=1, 3 </math>. |
− |
| |
− |
| |
− | == Solution 2 ==
| |
− |
| |
− | We claim that the only positive integers satisfying the given condition are <math>n=1</math> and <math>n=3.</math> These indeed work since <math>\sqrt{1}=1</math> and
| |
− | <cmath>\sqrt{2+\sqrt{3+\sqrt{1}}}=\sqrt{2+\sqrt{4}}=2</cmath>are both rational.
| |
− |
| |
− | First, we introduce some simplifying notation: Let <math>b_1,\ b_2,\ \ldots,\ b_m</math> be arbitrary positive integers. Then define
| |
− | <cmath>B_i=\sqrt{b_i + \sqrt{ b_{i-1} + \sqrt{\cdots + \sqrt{b_2+\sqrt{b_1}}}}}</cmath>for all <math>1 \le i \le m.</math>
| |
− |
| |
− | We start by showing three lemmas:
| |
− |
| |
− | Lemma 1: Let <math>k</math> be a positive integer. Then <math>\sqrt{k}</math> is rational if and only if <math>\sqrt{k}</math> is an integer.
| |
− |
| |
− | Proof: First, note that if <math>\sqrt{k}</math> is an integer, it must be rational. Now, suppose <math>\sqrt{k}</math> is rational; then we can write <math>k=\dfrac{x^2}{y^2}</math> for some relatively prime positive integers <math>x</math> and <math>y.</math> Hence, <math>k \cdot y^2=x^2,</math> so <math>y^2 \mid x^2.</math> Since <math>x</math> and <math>y</math> are relatively prime, it follows that <math>y \mid x,</math> which can only happen if <math>y=1.</math> Hence, we have <math>k=x^2,</math> which means that <math>\sqrt{k}</math> is an integer. <math>\blacksquare</math>
| |
− |
| |
− | Lemma 2: Let <math>b_1,\ b_2,\ \ldots,\ b_m</math> be arbitrary positive integers. Then <math>B_m</math> is rational if and only if <math>B_i</math> is an integer for all integers <math>1 \le i \le m.</math>
| |
− |
| |
− | Proof: We prove this by induction on <math>m.</math> The base case <math>m=1</math> is given by Lemma 1. Now, suppose the statement is true for <math>m-1.</math> If <math>B_i</math> is an integer for all <math>1 \le i \le m,</math> then <math>B_m</math> is an integer, so <math>B_m</math> is rational.
| |
− |
| |
− | Now, if <math>B_m</math> is rational, then <math>B_m^2-b_m=B_{m-1}</math> is also rational. By the inductive hypothesis, this is true if and only if <math>B_i</math> is an integer for all <math>1 \le i \le m-1.</math> In particular, <math>B_{m-1}</math> must be an integer, so <math>b_m+B_{m-1}</math> must also be an integer. Then, since <math>B_m</math> is rational by hypothesis, Lemma 1 tells us that <math>B_m=\sqrt{b_m+B_{m-1}}</math> must be an integer. Since we also know <math>B_i</math> is an integer for all <math>1 \le i \le m-1,</math> it follows that <math>B_i</math> is an integer for all <math>1 \le i \le m.</math> Hence, by induction, our statement is true for all <math>m,</math> as desired. <math>\blacksquare</math>
| |
− |
| |
− | Lemma 3: Suppose that <math>b_1,\ b_2,\ \ldots,\ b_m,\ k</math> are positive integers such that <math>b_1,\ b_2,\ \ldots,\ b_m \le k^2,</math> and <math>B_m</math> is rational. Then we have <math>B_m \le k.</math>
| |
− |
| |
− | Proof: Again, we prove this by induction on <math>m.</math> For the base case <math>m=1,</math> we know that since <math>b_1 \le k^2,</math> it follows that <math>\sqrt{b_1} \le k.</math>
| |
− |
| |
− | Now, suppose the statement is true for <math>m-1,</math> and suppose <math>b_1,\ b_2,\ \ldots,\ b_m,\ k</math> are positive integers such that <math>b_1,\ b_2,\ \ldots,\ b_m \le k^2</math> and <math>B_m</math> is rational. Then we know that <math>B_{m-1}=B_m^2-b_m</math> is also rational, so by the inductive hypothesis, we have <math>B_{m-1} \le k.</math> Adding <math>b_m</math> and then taking the square root of both sides, it follows that
| |
− | <cmath>B_m = \sqrt{b_m+B_{m-1}} \le \sqrt{k+b_m} \le \sqrt{k^2+k},</cmath>where the last inequality comes from the fact that <math>b_m \le k^2.</math> However, since Lemma 2 tells us that <math>B_m</math> must be an integer and <math>k < \sqrt{k^2+k} < k+1,</math> it follows that we actually have <math>B_m \le \sqrt{k^2}=k.</math> Hence, by induction, our statement is true for all <math>m,</math> as desired. <math>\blacksquare</math>
| |
− |
| |
− | Using these lemmas, we turn our attention to which positive integers <math>n</math> can satisfy the condition given in the problem. Similarly to our definition of <math>B_m</math> above, for a permutation <math>a_1,\ a_2,\ \ldots,\ a_n</math> of the numbers <math>1,\ 2,\ \ldots,\ n,</math> define
| |
− | <cmath>A_i=\sqrt{a_i + \sqrt{ a_{i-1} + \sqrt{\cdots + \sqrt{a_2+\sqrt{a_1}}}}}</cmath>for all <math>1 \le i \le n.</math>
| |
− |
| |
− | First, we claim that no <math>n \ge 5</math> satisfies the given condition. Suppose for the sake of contradiction that some <math>n \ge 5</math> did satisfy the condition. Then there must exist some integer <math>m \ge 2</math> such that <math>m^2+1 \le n \le (m+1)^2,</math> so there must be some <math>1 \le i \le n</math> such that <math>a_i=m^2+1.</math> If <math>i=1,</math> we know from Lemma 2 that <math>A_1=\sqrt{a_1}</math> is an integer, a contradiction. Hence, <math>i \neq 1.</math> Then, Lemma 3 tells us that since <math>a_i,\ a_{i-1},\ \ldots,\ a_1</math> are at most <math>n</math> and hence less than <math>(m+1)^2,</math> we have <math>A_i \le m+1.</math> This means that
| |
− | <cmath>A_i^2 = a_i + A_{i-1} = (m^2+1)+A_{i-1} \le (m+1)^2.</cmath>Furthermore, since Lemma 2 tells us <math>A_i</math> is an integer, <math>A_i^2</math> must be a perfect square. Since <math>m^2<A_i^2 \le (m+1)^2,</math> it follows that <math>A_i^2=(m+1)^2,</math> so <math>A_{i-1}=2m.</math> Then, as Lemma 3 tells us that <math>A_{i-1} \le m+1,</math> this means that <math>2m \le m+1</math> and hence <math>m \le 1,</math> contradicting the fact that <math>m \ge 2.</math> Therefore, no <math>n \ge 5</math> satisfies the given conditions.
| |
− |
| |
− | Hence, the only possible positive integers <math>n</math> that could satisfy the given condition are <math>n=1,\ 2,\ 3,</math> and <math>4.</math> We showed above that <math>n=1</math> and <math>n=3</math> do indeed satisfy the given condition, so it suffices to show that <math>n=2</math> and <math>n=4</math> do not. For <math>n=2,</math> we can verify using Lemma 1 that neither <math>\sqrt{1+\sqrt{2}}</math> nor <math>\sqrt{2+\sqrt{1}}</math> are rational, so this does not satisfy the given conditions.
| |
− |
| |
− | Now, suppose <math>n=4</math> satisfies the given conditions. First, suppose for the sake of contradiction that <math>a_1 \neq 4</math> and that <math>a_i=4</math> for some <math>2 \le i \le 4.</math> Then Lemma 3 tells us that <math>A_{i-1} \le 2,</math> so <math>A_i^2=a_i+A_{i-1}</math> must both be a perfect square and equal to either <math>5</math> or <math>6,</math> which is a contradiction. Hence, we must have <math>a_1=4.</math> By Lemma 2, it then follows that one of the numbers
| |
− | <cmath>\sqrt{1+\sqrt{4}},\ \sqrt{3+\sqrt{4}},\ \sqrt{1+\sqrt{2+\sqrt{4}}},\ \sqrt{3+\sqrt{2+\sqrt{4}}}</cmath>is an integer. However, we can verify using Lemma 1 that none of these are integers, so <math>n=4</math> does not satisfy the given conditions.
| |
− |
| |
− | This means that the only positive integers satisfying the given condition are <math>n=\boxed{1\text{ and }3},</math> as desired.
| |
| | | |
| {{alternate solutions}} | | {{alternate solutions}} |
Problem
(Serbia)
Find all positive integers such that there exists a permutation on the set for which
is a rational number.
Note: A permutation of the set is a one-to-one function of this set to itself.
Solution
The only solutions are and .
First, we will prove that for positive integers , the number is rational if and only if is an integer, for all integers . This follows from induction on . The case is trivial; now, suppose it holds for . Then if is an rational, than its square, must also be rational, which implies that is rational. But by the inductive hypothesis, must be an integer, so must also be an integer, which means that is also an integer, since it is rational. The result follows.
Next, we prove that if are positive integers such that and is rational, then .
This also comes by induction. The case is trivial. If our result holds for any , then
.
Since must be a perfect square, it can be at most , and the result follows.
Now we prove that no is a solution.
Suppose that there is some that is a solution. Than there exists some number such that . Since , . If , then we know since is not a square; and must be a perfect square, so we must have . But this is a contradiction.
We now have the cases to consider. The case is trivial. For it is easy to verify that neither nor is rational. For , we have the solution . We now consider. . First, suppose ; say . We have already shown that ; this means , a contradiction, since must be a perfect square. Thus . But here we have either is irrational, is irrational, is irrational, or is irrational. Thus there is no solution for and the only solutions are .
Alternate solutions are always welcome. If you have a different, elegant solution to this problem, please add it to this page.
Resources